Math, asked by suryanshumohansingh, 29 days ago

Please help me I will Mark you brainlist

Attachments:

Answers

Answered by ska9938278269
1

Answer:

the correct answer is a)-25 + 10

b)20 - 39

Step-by-step explanation:

brainlist pls

Similar questions